Find $limlimits_{nrightarrowinfty}intlimits_0^1f(x^n)dx$












4












$begingroup$



Let $f:[0,1]rightarrowmathbb{R}$ be a continuous function.




  1. Show that for each $epsilonin(0,1)$, $limlimits_{nrightarrowinfty}intlimits_0^{1-epsilon}f(x^n)dx=(1-epsilon)f(0)$

  2. Find $limlimits_{nrightarrowinfty}intlimits_{0}^{1}f(x^n)dx$.

    Hint: Start by explaining why $f$ is bounded.




For my Answer:,

I have done the part one by using the fact that
begin{align*}
left|intlimits_{0}^{1-epsilon}f(x^n)dx-(1-epsilon)f(0)right|leq intlimits_0^{1-epsilon}|f(x^n)-f(0)|dx
end{align*}

And with the continuity of $f$ at zero along with $x^nleq(1-epsilon)^nrightarrow0$

But for the part two what I can see is that:
begin{align*}
limlimits_{nrightarrowinfty}intlimits_{0}^{1}f(x^n)dx &=limlimits_{nrightarrowinfty}limlimits_{epsilonrightarrow0}intlimits_{0}^{1-epsilon}f(x^n)dx
end{align*}

But there after if I need to make use of part (1) then I have to interchange the limits. So is it possible. If so, I would like to know what are the conditions we need to have for such an interchange.

Moreover I would like a feedback on the hint given. (Why is it given?)










share|cite|improve this question











$endgroup$

















    4












    $begingroup$



    Let $f:[0,1]rightarrowmathbb{R}$ be a continuous function.




    1. Show that for each $epsilonin(0,1)$, $limlimits_{nrightarrowinfty}intlimits_0^{1-epsilon}f(x^n)dx=(1-epsilon)f(0)$

    2. Find $limlimits_{nrightarrowinfty}intlimits_{0}^{1}f(x^n)dx$.

      Hint: Start by explaining why $f$ is bounded.




    For my Answer:,

    I have done the part one by using the fact that
    begin{align*}
    left|intlimits_{0}^{1-epsilon}f(x^n)dx-(1-epsilon)f(0)right|leq intlimits_0^{1-epsilon}|f(x^n)-f(0)|dx
    end{align*}

    And with the continuity of $f$ at zero along with $x^nleq(1-epsilon)^nrightarrow0$

    But for the part two what I can see is that:
    begin{align*}
    limlimits_{nrightarrowinfty}intlimits_{0}^{1}f(x^n)dx &=limlimits_{nrightarrowinfty}limlimits_{epsilonrightarrow0}intlimits_{0}^{1-epsilon}f(x^n)dx
    end{align*}

    But there after if I need to make use of part (1) then I have to interchange the limits. So is it possible. If so, I would like to know what are the conditions we need to have for such an interchange.

    Moreover I would like a feedback on the hint given. (Why is it given?)










    share|cite|improve this question











    $endgroup$















      4












      4








      4


      3



      $begingroup$



      Let $f:[0,1]rightarrowmathbb{R}$ be a continuous function.




      1. Show that for each $epsilonin(0,1)$, $limlimits_{nrightarrowinfty}intlimits_0^{1-epsilon}f(x^n)dx=(1-epsilon)f(0)$

      2. Find $limlimits_{nrightarrowinfty}intlimits_{0}^{1}f(x^n)dx$.

        Hint: Start by explaining why $f$ is bounded.




      For my Answer:,

      I have done the part one by using the fact that
      begin{align*}
      left|intlimits_{0}^{1-epsilon}f(x^n)dx-(1-epsilon)f(0)right|leq intlimits_0^{1-epsilon}|f(x^n)-f(0)|dx
      end{align*}

      And with the continuity of $f$ at zero along with $x^nleq(1-epsilon)^nrightarrow0$

      But for the part two what I can see is that:
      begin{align*}
      limlimits_{nrightarrowinfty}intlimits_{0}^{1}f(x^n)dx &=limlimits_{nrightarrowinfty}limlimits_{epsilonrightarrow0}intlimits_{0}^{1-epsilon}f(x^n)dx
      end{align*}

      But there after if I need to make use of part (1) then I have to interchange the limits. So is it possible. If so, I would like to know what are the conditions we need to have for such an interchange.

      Moreover I would like a feedback on the hint given. (Why is it given?)










      share|cite|improve this question











      $endgroup$





      Let $f:[0,1]rightarrowmathbb{R}$ be a continuous function.




      1. Show that for each $epsilonin(0,1)$, $limlimits_{nrightarrowinfty}intlimits_0^{1-epsilon}f(x^n)dx=(1-epsilon)f(0)$

      2. Find $limlimits_{nrightarrowinfty}intlimits_{0}^{1}f(x^n)dx$.

        Hint: Start by explaining why $f$ is bounded.




      For my Answer:,

      I have done the part one by using the fact that
      begin{align*}
      left|intlimits_{0}^{1-epsilon}f(x^n)dx-(1-epsilon)f(0)right|leq intlimits_0^{1-epsilon}|f(x^n)-f(0)|dx
      end{align*}

      And with the continuity of $f$ at zero along with $x^nleq(1-epsilon)^nrightarrow0$

      But for the part two what I can see is that:
      begin{align*}
      limlimits_{nrightarrowinfty}intlimits_{0}^{1}f(x^n)dx &=limlimits_{nrightarrowinfty}limlimits_{epsilonrightarrow0}intlimits_{0}^{1-epsilon}f(x^n)dx
      end{align*}

      But there after if I need to make use of part (1) then I have to interchange the limits. So is it possible. If so, I would like to know what are the conditions we need to have for such an interchange.

      Moreover I would like a feedback on the hint given. (Why is it given?)







      real-analysis riemann-integration






      share|cite|improve this question















      share|cite|improve this question













      share|cite|improve this question




      share|cite|improve this question








      edited Jan 6 at 22:35









      Did

      247k23222458




      247k23222458










      asked Jan 6 at 22:30









      DD90DD90

      2628




      2628






















          4 Answers
          4






          active

          oldest

          votes


















          2












          $begingroup$

          Okay, so please let me state an analysis principle that will be very useful in numerous problems.



          Interversion is painful.



          Either there is a straightforward (« logically tautological », that is, that does not require any analysis) argument for doing it, or it requires thinking the whole reasoning through with more elaborate arguments (such as some form of uniform convergence).



          So now, let $M$ be a bound for $f$. Let $epsilon >0$. I suggest that you prove that



          $$left|int_0^1{f(x^n)}-f(0)right| leq left|int_0^{1-epsilon}{f(x^n)}-(1-epsilon)f(0)right| + Mepsilon + epsilon |f(0)|. $$



          Thus for every $n$ large enough $$left|int_0^1{f(x^n)}-f(0)right| leq 3Mepsilon$$.






          share|cite|improve this answer









          $endgroup$













          • $begingroup$
            Thank you @Mindlack.So this means we are proving the limit definition for this one as well. ? Also I think if we choose the interval as $(0,frac{epsilon}{3})$ we would end up in epsilon alone in the RHS
            $endgroup$
            – DD90
            Jan 6 at 23:14






          • 1




            $begingroup$
            Yes, unless you can use some more advanced machinery (double-limit theorems, uniform convergence and stuff), you have to prove the limit using $epsilon$. On another note: if you wanted to have some $epsilon$ in the final estimate, you would have to take the integral to $1-frac{epsilon}{3M+1}$.
            $endgroup$
            – Mindlack
            Jan 6 at 23:18



















          1












          $begingroup$

          As $f$ is continuous at $0$, given $c>0$, there exists $delta>0$ such that $|f(x)-f(0)|<c$ whenever $|x|<delta$. So choose $n_0$ such that $(1-epsilon)^{n_0}<delta$. Then, for any $x<1-epsilon$ and $ngeq n_0$, we have $x^n<delta$; then $|f(x^n)-f(0)|<c$. Then
          $$
          (1-epsilon)(f(0)-c)=int_0^{(1-epsilon)}(f(0)-c),dxleqint_0^{1-epsilon}f(x^n),dxleq int_0^{(1-epsilon)}(f(0)+c),dx=(1-epsilon)(f(0)+c).
          $$

          As we can do this for all $c>0$, we get
          $$
          int_0^{1-epsilon}f(x^n),dx=(1-epsilon)f(0)
          $$



          For the second part, because $f$ is continuous and $[0,1]$ is compact, $f$ is bounded, say $aleq f(x)leq b$ for all $x$. Then, for any $epsilon>0$,
          $$
          int_0^1f(x^n),dx=int_0^{(1-epsilon)}f(x^n),dx+int_{(1-epsilon)}^1f(x^n),dx.
          $$

          For the second integral,
          $$
          epsilon, aleq int_{(1-epsilon)}^1f(x^n),dxleq epsilon, b.
          $$

          Then
          $$
          limsup_nint_0^1f(x^n),dx= (1-epsilon)f(0)+int_{(1-epsilon)}^1f(x^n),dxleq(1-epsilon)f(0)+epsilon,b.
          $$

          As this works for all $epsilon>0$,
          $$tag1
          limsup_nint_0^1f(x^n),dxleq f(0).
          $$

          Similarly,
          $$
          liminf_nint_0^1f(x^n),dx= (1-epsilon)f(0)+int_{(1-epsilon)}^1f(x^n),dxgeq(1-epsilon)f(0)+epsilon,a.
          $$

          As this works for all $epsilon>0$,
          $$tag2
          liminf_nint_0^1f(x^n),dxgeq f(0).
          $$

          Now $(1)$ and $(2)$ together imply that that $lim_nint_0^1f(x^n),dx$ exists and that
          $$
          lim_nint_0^1f(x^n),dx=f(0).
          $$






          share|cite|improve this answer









          $endgroup$





















            0












            $begingroup$

            So you managed to show that for each $varepsilonin(0,1)$ you get:
            $$ limlimits_{ntoinfty}intlimits_0^{1-varepsilon}f(x^n),{rm d}x=(1-varepsilon)f(0) $$
            So for each $varepsilonin(0,1)$ you can now look at the following:
            $$ intlimits_0^1f(x^n),{rm d}x=intlimits_0^{1-varepsilon}f(x^n),{rm d}x+intlimits_{1-varepsilon}^1f(x^n),{rm d}x$$
            Since you know that $f$ is bounded (a continuous function on a closed interval) you know there exists some $M>0$ such that:
            $$ forall xin(0,1),-Mleq f(x)leq M $$
            So you can show that for second interval you get:
            $$ -Mvarepsilonleqintlimits_{1-varepsilon}^1f(x^n){rm d}xleq Mvarepsilon $$
            Or in other words:
            $$ intlimits_0^{1-varepsilon}f(x^n),{rm d}x-Mvarepsilonleq intlimits_0^1f(x^n),{rm d}xleq intlimits_0^{1-varepsilon}f(x^n){rm d}x+Mvarepsilon $$
            Taking the limits on all sides as $ntoinfty$ you get using the first part:
            $$(1-varepsilon)f(0)-Mvarepsilonleqlimlimits_{ntoinfty}intlimits_0^1f(x^n),{rm d}xleq(1-varepsilon)f(0)+Mvarepsilon $$
            Lastly, taking the limit $varepsilonto 0$ gives you the result:
            $$ limlimits_{ntoinfty}intlimits_0^1f(x^n),{rm d}x=f(0) $$






            share|cite|improve this answer









            $endgroup$





















              0












              $begingroup$

              Another approach uses the mean value theorem for integrals. Indeed, set $g_n(x)=f(x^n).$ Then, there is a sequence $(c_n)subseteq (0,1-epsilon)$ such that $intlimits_0^{1-epsilon}g_n(x)dx=g_n(c_n)(1-epsilon)=f(c^n_n)(1-epsilon).$ Then, $c_n^nto 0$ as $nto infty$ and so $f(c_n^n)to f(0).$



              For the second part, note that $int_0^1f(x^n),dx=int_0^{1-epsilon}f(x^n),dx+int_{1-epsilon}^1f(x^n),dx,$ so we just need to consider the second integral. But as $g_n$ is bounded, we can apply the dominated convergence theorem to write $lim int_{1-epsilon}^1f(x^n),dx=int_{1-epsilon}^1f(0)dx=epsilon f(0)$.



              Therefore, $lim int_0^1f(x^n),dx= f(0).$






              share|cite|improve this answer











              $endgroup$













              • $begingroup$
                why the downvote? since $(c_n)subseteq (0,1-epsilon), limsup c_n^nto 0$ and since $f$ is continuous, $f(c_n^n)to 0$.
                $endgroup$
                – Matematleta
                Jan 6 at 23:16











              Your Answer





              StackExchange.ifUsing("editor", function () {
              return StackExchange.using("mathjaxEditing", function () {
              StackExchange.MarkdownEditor.creationCallbacks.add(function (editor, postfix) {
              StackExchange.mathjaxEditing.prepareWmdForMathJax(editor, postfix, [["$", "$"], ["\\(","\\)"]]);
              });
              });
              }, "mathjax-editing");

              StackExchange.ready(function() {
              var channelOptions = {
              tags: "".split(" "),
              id: "69"
              };
              initTagRenderer("".split(" "), "".split(" "), channelOptions);

              StackExchange.using("externalEditor", function() {
              // Have to fire editor after snippets, if snippets enabled
              if (StackExchange.settings.snippets.snippetsEnabled) {
              StackExchange.using("snippets", function() {
              createEditor();
              });
              }
              else {
              createEditor();
              }
              });

              function createEditor() {
              StackExchange.prepareEditor({
              heartbeatType: 'answer',
              autoActivateHeartbeat: false,
              convertImagesToLinks: true,
              noModals: true,
              showLowRepImageUploadWarning: true,
              reputationToPostImages: 10,
              bindNavPrevention: true,
              postfix: "",
              imageUploader: {
              brandingHtml: "Powered by u003ca class="icon-imgur-white" href="https://imgur.com/"u003eu003c/au003e",
              contentPolicyHtml: "User contributions licensed under u003ca href="https://creativecommons.org/licenses/by-sa/3.0/"u003ecc by-sa 3.0 with attribution requiredu003c/au003e u003ca href="https://stackoverflow.com/legal/content-policy"u003e(content policy)u003c/au003e",
              allowUrls: true
              },
              noCode: true, onDemand: true,
              discardSelector: ".discard-answer"
              ,immediatelyShowMarkdownHelp:true
              });


              }
              });














              draft saved

              draft discarded


















              StackExchange.ready(
              function () {
              StackExchange.openid.initPostLogin('.new-post-login', 'https%3a%2f%2fmath.stackexchange.com%2fquestions%2f3064458%2ffind-lim-limits-n-rightarrow-infty-int-limits-01fxndx%23new-answer', 'question_page');
              }
              );

              Post as a guest















              Required, but never shown

























              4 Answers
              4






              active

              oldest

              votes








              4 Answers
              4






              active

              oldest

              votes









              active

              oldest

              votes






              active

              oldest

              votes









              2












              $begingroup$

              Okay, so please let me state an analysis principle that will be very useful in numerous problems.



              Interversion is painful.



              Either there is a straightforward (« logically tautological », that is, that does not require any analysis) argument for doing it, or it requires thinking the whole reasoning through with more elaborate arguments (such as some form of uniform convergence).



              So now, let $M$ be a bound for $f$. Let $epsilon >0$. I suggest that you prove that



              $$left|int_0^1{f(x^n)}-f(0)right| leq left|int_0^{1-epsilon}{f(x^n)}-(1-epsilon)f(0)right| + Mepsilon + epsilon |f(0)|. $$



              Thus for every $n$ large enough $$left|int_0^1{f(x^n)}-f(0)right| leq 3Mepsilon$$.






              share|cite|improve this answer









              $endgroup$













              • $begingroup$
                Thank you @Mindlack.So this means we are proving the limit definition for this one as well. ? Also I think if we choose the interval as $(0,frac{epsilon}{3})$ we would end up in epsilon alone in the RHS
                $endgroup$
                – DD90
                Jan 6 at 23:14






              • 1




                $begingroup$
                Yes, unless you can use some more advanced machinery (double-limit theorems, uniform convergence and stuff), you have to prove the limit using $epsilon$. On another note: if you wanted to have some $epsilon$ in the final estimate, you would have to take the integral to $1-frac{epsilon}{3M+1}$.
                $endgroup$
                – Mindlack
                Jan 6 at 23:18
















              2












              $begingroup$

              Okay, so please let me state an analysis principle that will be very useful in numerous problems.



              Interversion is painful.



              Either there is a straightforward (« logically tautological », that is, that does not require any analysis) argument for doing it, or it requires thinking the whole reasoning through with more elaborate arguments (such as some form of uniform convergence).



              So now, let $M$ be a bound for $f$. Let $epsilon >0$. I suggest that you prove that



              $$left|int_0^1{f(x^n)}-f(0)right| leq left|int_0^{1-epsilon}{f(x^n)}-(1-epsilon)f(0)right| + Mepsilon + epsilon |f(0)|. $$



              Thus for every $n$ large enough $$left|int_0^1{f(x^n)}-f(0)right| leq 3Mepsilon$$.






              share|cite|improve this answer









              $endgroup$













              • $begingroup$
                Thank you @Mindlack.So this means we are proving the limit definition for this one as well. ? Also I think if we choose the interval as $(0,frac{epsilon}{3})$ we would end up in epsilon alone in the RHS
                $endgroup$
                – DD90
                Jan 6 at 23:14






              • 1




                $begingroup$
                Yes, unless you can use some more advanced machinery (double-limit theorems, uniform convergence and stuff), you have to prove the limit using $epsilon$. On another note: if you wanted to have some $epsilon$ in the final estimate, you would have to take the integral to $1-frac{epsilon}{3M+1}$.
                $endgroup$
                – Mindlack
                Jan 6 at 23:18














              2












              2








              2





              $begingroup$

              Okay, so please let me state an analysis principle that will be very useful in numerous problems.



              Interversion is painful.



              Either there is a straightforward (« logically tautological », that is, that does not require any analysis) argument for doing it, or it requires thinking the whole reasoning through with more elaborate arguments (such as some form of uniform convergence).



              So now, let $M$ be a bound for $f$. Let $epsilon >0$. I suggest that you prove that



              $$left|int_0^1{f(x^n)}-f(0)right| leq left|int_0^{1-epsilon}{f(x^n)}-(1-epsilon)f(0)right| + Mepsilon + epsilon |f(0)|. $$



              Thus for every $n$ large enough $$left|int_0^1{f(x^n)}-f(0)right| leq 3Mepsilon$$.






              share|cite|improve this answer









              $endgroup$



              Okay, so please let me state an analysis principle that will be very useful in numerous problems.



              Interversion is painful.



              Either there is a straightforward (« logically tautological », that is, that does not require any analysis) argument for doing it, or it requires thinking the whole reasoning through with more elaborate arguments (such as some form of uniform convergence).



              So now, let $M$ be a bound for $f$. Let $epsilon >0$. I suggest that you prove that



              $$left|int_0^1{f(x^n)}-f(0)right| leq left|int_0^{1-epsilon}{f(x^n)}-(1-epsilon)f(0)right| + Mepsilon + epsilon |f(0)|. $$



              Thus for every $n$ large enough $$left|int_0^1{f(x^n)}-f(0)right| leq 3Mepsilon$$.







              share|cite|improve this answer












              share|cite|improve this answer



              share|cite|improve this answer










              answered Jan 6 at 22:44









              MindlackMindlack

              2,86717




              2,86717












              • $begingroup$
                Thank you @Mindlack.So this means we are proving the limit definition for this one as well. ? Also I think if we choose the interval as $(0,frac{epsilon}{3})$ we would end up in epsilon alone in the RHS
                $endgroup$
                – DD90
                Jan 6 at 23:14






              • 1




                $begingroup$
                Yes, unless you can use some more advanced machinery (double-limit theorems, uniform convergence and stuff), you have to prove the limit using $epsilon$. On another note: if you wanted to have some $epsilon$ in the final estimate, you would have to take the integral to $1-frac{epsilon}{3M+1}$.
                $endgroup$
                – Mindlack
                Jan 6 at 23:18


















              • $begingroup$
                Thank you @Mindlack.So this means we are proving the limit definition for this one as well. ? Also I think if we choose the interval as $(0,frac{epsilon}{3})$ we would end up in epsilon alone in the RHS
                $endgroup$
                – DD90
                Jan 6 at 23:14






              • 1




                $begingroup$
                Yes, unless you can use some more advanced machinery (double-limit theorems, uniform convergence and stuff), you have to prove the limit using $epsilon$. On another note: if you wanted to have some $epsilon$ in the final estimate, you would have to take the integral to $1-frac{epsilon}{3M+1}$.
                $endgroup$
                – Mindlack
                Jan 6 at 23:18
















              $begingroup$
              Thank you @Mindlack.So this means we are proving the limit definition for this one as well. ? Also I think if we choose the interval as $(0,frac{epsilon}{3})$ we would end up in epsilon alone in the RHS
              $endgroup$
              – DD90
              Jan 6 at 23:14




              $begingroup$
              Thank you @Mindlack.So this means we are proving the limit definition for this one as well. ? Also I think if we choose the interval as $(0,frac{epsilon}{3})$ we would end up in epsilon alone in the RHS
              $endgroup$
              – DD90
              Jan 6 at 23:14




              1




              1




              $begingroup$
              Yes, unless you can use some more advanced machinery (double-limit theorems, uniform convergence and stuff), you have to prove the limit using $epsilon$. On another note: if you wanted to have some $epsilon$ in the final estimate, you would have to take the integral to $1-frac{epsilon}{3M+1}$.
              $endgroup$
              – Mindlack
              Jan 6 at 23:18




              $begingroup$
              Yes, unless you can use some more advanced machinery (double-limit theorems, uniform convergence and stuff), you have to prove the limit using $epsilon$. On another note: if you wanted to have some $epsilon$ in the final estimate, you would have to take the integral to $1-frac{epsilon}{3M+1}$.
              $endgroup$
              – Mindlack
              Jan 6 at 23:18











              1












              $begingroup$

              As $f$ is continuous at $0$, given $c>0$, there exists $delta>0$ such that $|f(x)-f(0)|<c$ whenever $|x|<delta$. So choose $n_0$ such that $(1-epsilon)^{n_0}<delta$. Then, for any $x<1-epsilon$ and $ngeq n_0$, we have $x^n<delta$; then $|f(x^n)-f(0)|<c$. Then
              $$
              (1-epsilon)(f(0)-c)=int_0^{(1-epsilon)}(f(0)-c),dxleqint_0^{1-epsilon}f(x^n),dxleq int_0^{(1-epsilon)}(f(0)+c),dx=(1-epsilon)(f(0)+c).
              $$

              As we can do this for all $c>0$, we get
              $$
              int_0^{1-epsilon}f(x^n),dx=(1-epsilon)f(0)
              $$



              For the second part, because $f$ is continuous and $[0,1]$ is compact, $f$ is bounded, say $aleq f(x)leq b$ for all $x$. Then, for any $epsilon>0$,
              $$
              int_0^1f(x^n),dx=int_0^{(1-epsilon)}f(x^n),dx+int_{(1-epsilon)}^1f(x^n),dx.
              $$

              For the second integral,
              $$
              epsilon, aleq int_{(1-epsilon)}^1f(x^n),dxleq epsilon, b.
              $$

              Then
              $$
              limsup_nint_0^1f(x^n),dx= (1-epsilon)f(0)+int_{(1-epsilon)}^1f(x^n),dxleq(1-epsilon)f(0)+epsilon,b.
              $$

              As this works for all $epsilon>0$,
              $$tag1
              limsup_nint_0^1f(x^n),dxleq f(0).
              $$

              Similarly,
              $$
              liminf_nint_0^1f(x^n),dx= (1-epsilon)f(0)+int_{(1-epsilon)}^1f(x^n),dxgeq(1-epsilon)f(0)+epsilon,a.
              $$

              As this works for all $epsilon>0$,
              $$tag2
              liminf_nint_0^1f(x^n),dxgeq f(0).
              $$

              Now $(1)$ and $(2)$ together imply that that $lim_nint_0^1f(x^n),dx$ exists and that
              $$
              lim_nint_0^1f(x^n),dx=f(0).
              $$






              share|cite|improve this answer









              $endgroup$


















                1












                $begingroup$

                As $f$ is continuous at $0$, given $c>0$, there exists $delta>0$ such that $|f(x)-f(0)|<c$ whenever $|x|<delta$. So choose $n_0$ such that $(1-epsilon)^{n_0}<delta$. Then, for any $x<1-epsilon$ and $ngeq n_0$, we have $x^n<delta$; then $|f(x^n)-f(0)|<c$. Then
                $$
                (1-epsilon)(f(0)-c)=int_0^{(1-epsilon)}(f(0)-c),dxleqint_0^{1-epsilon}f(x^n),dxleq int_0^{(1-epsilon)}(f(0)+c),dx=(1-epsilon)(f(0)+c).
                $$

                As we can do this for all $c>0$, we get
                $$
                int_0^{1-epsilon}f(x^n),dx=(1-epsilon)f(0)
                $$



                For the second part, because $f$ is continuous and $[0,1]$ is compact, $f$ is bounded, say $aleq f(x)leq b$ for all $x$. Then, for any $epsilon>0$,
                $$
                int_0^1f(x^n),dx=int_0^{(1-epsilon)}f(x^n),dx+int_{(1-epsilon)}^1f(x^n),dx.
                $$

                For the second integral,
                $$
                epsilon, aleq int_{(1-epsilon)}^1f(x^n),dxleq epsilon, b.
                $$

                Then
                $$
                limsup_nint_0^1f(x^n),dx= (1-epsilon)f(0)+int_{(1-epsilon)}^1f(x^n),dxleq(1-epsilon)f(0)+epsilon,b.
                $$

                As this works for all $epsilon>0$,
                $$tag1
                limsup_nint_0^1f(x^n),dxleq f(0).
                $$

                Similarly,
                $$
                liminf_nint_0^1f(x^n),dx= (1-epsilon)f(0)+int_{(1-epsilon)}^1f(x^n),dxgeq(1-epsilon)f(0)+epsilon,a.
                $$

                As this works for all $epsilon>0$,
                $$tag2
                liminf_nint_0^1f(x^n),dxgeq f(0).
                $$

                Now $(1)$ and $(2)$ together imply that that $lim_nint_0^1f(x^n),dx$ exists and that
                $$
                lim_nint_0^1f(x^n),dx=f(0).
                $$






                share|cite|improve this answer









                $endgroup$
















                  1












                  1








                  1





                  $begingroup$

                  As $f$ is continuous at $0$, given $c>0$, there exists $delta>0$ such that $|f(x)-f(0)|<c$ whenever $|x|<delta$. So choose $n_0$ such that $(1-epsilon)^{n_0}<delta$. Then, for any $x<1-epsilon$ and $ngeq n_0$, we have $x^n<delta$; then $|f(x^n)-f(0)|<c$. Then
                  $$
                  (1-epsilon)(f(0)-c)=int_0^{(1-epsilon)}(f(0)-c),dxleqint_0^{1-epsilon}f(x^n),dxleq int_0^{(1-epsilon)}(f(0)+c),dx=(1-epsilon)(f(0)+c).
                  $$

                  As we can do this for all $c>0$, we get
                  $$
                  int_0^{1-epsilon}f(x^n),dx=(1-epsilon)f(0)
                  $$



                  For the second part, because $f$ is continuous and $[0,1]$ is compact, $f$ is bounded, say $aleq f(x)leq b$ for all $x$. Then, for any $epsilon>0$,
                  $$
                  int_0^1f(x^n),dx=int_0^{(1-epsilon)}f(x^n),dx+int_{(1-epsilon)}^1f(x^n),dx.
                  $$

                  For the second integral,
                  $$
                  epsilon, aleq int_{(1-epsilon)}^1f(x^n),dxleq epsilon, b.
                  $$

                  Then
                  $$
                  limsup_nint_0^1f(x^n),dx= (1-epsilon)f(0)+int_{(1-epsilon)}^1f(x^n),dxleq(1-epsilon)f(0)+epsilon,b.
                  $$

                  As this works for all $epsilon>0$,
                  $$tag1
                  limsup_nint_0^1f(x^n),dxleq f(0).
                  $$

                  Similarly,
                  $$
                  liminf_nint_0^1f(x^n),dx= (1-epsilon)f(0)+int_{(1-epsilon)}^1f(x^n),dxgeq(1-epsilon)f(0)+epsilon,a.
                  $$

                  As this works for all $epsilon>0$,
                  $$tag2
                  liminf_nint_0^1f(x^n),dxgeq f(0).
                  $$

                  Now $(1)$ and $(2)$ together imply that that $lim_nint_0^1f(x^n),dx$ exists and that
                  $$
                  lim_nint_0^1f(x^n),dx=f(0).
                  $$






                  share|cite|improve this answer









                  $endgroup$



                  As $f$ is continuous at $0$, given $c>0$, there exists $delta>0$ such that $|f(x)-f(0)|<c$ whenever $|x|<delta$. So choose $n_0$ such that $(1-epsilon)^{n_0}<delta$. Then, for any $x<1-epsilon$ and $ngeq n_0$, we have $x^n<delta$; then $|f(x^n)-f(0)|<c$. Then
                  $$
                  (1-epsilon)(f(0)-c)=int_0^{(1-epsilon)}(f(0)-c),dxleqint_0^{1-epsilon}f(x^n),dxleq int_0^{(1-epsilon)}(f(0)+c),dx=(1-epsilon)(f(0)+c).
                  $$

                  As we can do this for all $c>0$, we get
                  $$
                  int_0^{1-epsilon}f(x^n),dx=(1-epsilon)f(0)
                  $$



                  For the second part, because $f$ is continuous and $[0,1]$ is compact, $f$ is bounded, say $aleq f(x)leq b$ for all $x$. Then, for any $epsilon>0$,
                  $$
                  int_0^1f(x^n),dx=int_0^{(1-epsilon)}f(x^n),dx+int_{(1-epsilon)}^1f(x^n),dx.
                  $$

                  For the second integral,
                  $$
                  epsilon, aleq int_{(1-epsilon)}^1f(x^n),dxleq epsilon, b.
                  $$

                  Then
                  $$
                  limsup_nint_0^1f(x^n),dx= (1-epsilon)f(0)+int_{(1-epsilon)}^1f(x^n),dxleq(1-epsilon)f(0)+epsilon,b.
                  $$

                  As this works for all $epsilon>0$,
                  $$tag1
                  limsup_nint_0^1f(x^n),dxleq f(0).
                  $$

                  Similarly,
                  $$
                  liminf_nint_0^1f(x^n),dx= (1-epsilon)f(0)+int_{(1-epsilon)}^1f(x^n),dxgeq(1-epsilon)f(0)+epsilon,a.
                  $$

                  As this works for all $epsilon>0$,
                  $$tag2
                  liminf_nint_0^1f(x^n),dxgeq f(0).
                  $$

                  Now $(1)$ and $(2)$ together imply that that $lim_nint_0^1f(x^n),dx$ exists and that
                  $$
                  lim_nint_0^1f(x^n),dx=f(0).
                  $$







                  share|cite|improve this answer












                  share|cite|improve this answer



                  share|cite|improve this answer










                  answered Jan 6 at 22:56









                  Martin ArgeramiMartin Argerami

                  125k1177177




                  125k1177177























                      0












                      $begingroup$

                      So you managed to show that for each $varepsilonin(0,1)$ you get:
                      $$ limlimits_{ntoinfty}intlimits_0^{1-varepsilon}f(x^n),{rm d}x=(1-varepsilon)f(0) $$
                      So for each $varepsilonin(0,1)$ you can now look at the following:
                      $$ intlimits_0^1f(x^n),{rm d}x=intlimits_0^{1-varepsilon}f(x^n),{rm d}x+intlimits_{1-varepsilon}^1f(x^n),{rm d}x$$
                      Since you know that $f$ is bounded (a continuous function on a closed interval) you know there exists some $M>0$ such that:
                      $$ forall xin(0,1),-Mleq f(x)leq M $$
                      So you can show that for second interval you get:
                      $$ -Mvarepsilonleqintlimits_{1-varepsilon}^1f(x^n){rm d}xleq Mvarepsilon $$
                      Or in other words:
                      $$ intlimits_0^{1-varepsilon}f(x^n),{rm d}x-Mvarepsilonleq intlimits_0^1f(x^n),{rm d}xleq intlimits_0^{1-varepsilon}f(x^n){rm d}x+Mvarepsilon $$
                      Taking the limits on all sides as $ntoinfty$ you get using the first part:
                      $$(1-varepsilon)f(0)-Mvarepsilonleqlimlimits_{ntoinfty}intlimits_0^1f(x^n),{rm d}xleq(1-varepsilon)f(0)+Mvarepsilon $$
                      Lastly, taking the limit $varepsilonto 0$ gives you the result:
                      $$ limlimits_{ntoinfty}intlimits_0^1f(x^n),{rm d}x=f(0) $$






                      share|cite|improve this answer









                      $endgroup$


















                        0












                        $begingroup$

                        So you managed to show that for each $varepsilonin(0,1)$ you get:
                        $$ limlimits_{ntoinfty}intlimits_0^{1-varepsilon}f(x^n),{rm d}x=(1-varepsilon)f(0) $$
                        So for each $varepsilonin(0,1)$ you can now look at the following:
                        $$ intlimits_0^1f(x^n),{rm d}x=intlimits_0^{1-varepsilon}f(x^n),{rm d}x+intlimits_{1-varepsilon}^1f(x^n),{rm d}x$$
                        Since you know that $f$ is bounded (a continuous function on a closed interval) you know there exists some $M>0$ such that:
                        $$ forall xin(0,1),-Mleq f(x)leq M $$
                        So you can show that for second interval you get:
                        $$ -Mvarepsilonleqintlimits_{1-varepsilon}^1f(x^n){rm d}xleq Mvarepsilon $$
                        Or in other words:
                        $$ intlimits_0^{1-varepsilon}f(x^n),{rm d}x-Mvarepsilonleq intlimits_0^1f(x^n),{rm d}xleq intlimits_0^{1-varepsilon}f(x^n){rm d}x+Mvarepsilon $$
                        Taking the limits on all sides as $ntoinfty$ you get using the first part:
                        $$(1-varepsilon)f(0)-Mvarepsilonleqlimlimits_{ntoinfty}intlimits_0^1f(x^n),{rm d}xleq(1-varepsilon)f(0)+Mvarepsilon $$
                        Lastly, taking the limit $varepsilonto 0$ gives you the result:
                        $$ limlimits_{ntoinfty}intlimits_0^1f(x^n),{rm d}x=f(0) $$






                        share|cite|improve this answer









                        $endgroup$
















                          0












                          0








                          0





                          $begingroup$

                          So you managed to show that for each $varepsilonin(0,1)$ you get:
                          $$ limlimits_{ntoinfty}intlimits_0^{1-varepsilon}f(x^n),{rm d}x=(1-varepsilon)f(0) $$
                          So for each $varepsilonin(0,1)$ you can now look at the following:
                          $$ intlimits_0^1f(x^n),{rm d}x=intlimits_0^{1-varepsilon}f(x^n),{rm d}x+intlimits_{1-varepsilon}^1f(x^n),{rm d}x$$
                          Since you know that $f$ is bounded (a continuous function on a closed interval) you know there exists some $M>0$ such that:
                          $$ forall xin(0,1),-Mleq f(x)leq M $$
                          So you can show that for second interval you get:
                          $$ -Mvarepsilonleqintlimits_{1-varepsilon}^1f(x^n){rm d}xleq Mvarepsilon $$
                          Or in other words:
                          $$ intlimits_0^{1-varepsilon}f(x^n),{rm d}x-Mvarepsilonleq intlimits_0^1f(x^n),{rm d}xleq intlimits_0^{1-varepsilon}f(x^n){rm d}x+Mvarepsilon $$
                          Taking the limits on all sides as $ntoinfty$ you get using the first part:
                          $$(1-varepsilon)f(0)-Mvarepsilonleqlimlimits_{ntoinfty}intlimits_0^1f(x^n),{rm d}xleq(1-varepsilon)f(0)+Mvarepsilon $$
                          Lastly, taking the limit $varepsilonto 0$ gives you the result:
                          $$ limlimits_{ntoinfty}intlimits_0^1f(x^n),{rm d}x=f(0) $$






                          share|cite|improve this answer









                          $endgroup$



                          So you managed to show that for each $varepsilonin(0,1)$ you get:
                          $$ limlimits_{ntoinfty}intlimits_0^{1-varepsilon}f(x^n),{rm d}x=(1-varepsilon)f(0) $$
                          So for each $varepsilonin(0,1)$ you can now look at the following:
                          $$ intlimits_0^1f(x^n),{rm d}x=intlimits_0^{1-varepsilon}f(x^n),{rm d}x+intlimits_{1-varepsilon}^1f(x^n),{rm d}x$$
                          Since you know that $f$ is bounded (a continuous function on a closed interval) you know there exists some $M>0$ such that:
                          $$ forall xin(0,1),-Mleq f(x)leq M $$
                          So you can show that for second interval you get:
                          $$ -Mvarepsilonleqintlimits_{1-varepsilon}^1f(x^n){rm d}xleq Mvarepsilon $$
                          Or in other words:
                          $$ intlimits_0^{1-varepsilon}f(x^n),{rm d}x-Mvarepsilonleq intlimits_0^1f(x^n),{rm d}xleq intlimits_0^{1-varepsilon}f(x^n){rm d}x+Mvarepsilon $$
                          Taking the limits on all sides as $ntoinfty$ you get using the first part:
                          $$(1-varepsilon)f(0)-Mvarepsilonleqlimlimits_{ntoinfty}intlimits_0^1f(x^n),{rm d}xleq(1-varepsilon)f(0)+Mvarepsilon $$
                          Lastly, taking the limit $varepsilonto 0$ gives you the result:
                          $$ limlimits_{ntoinfty}intlimits_0^1f(x^n),{rm d}x=f(0) $$







                          share|cite|improve this answer












                          share|cite|improve this answer



                          share|cite|improve this answer










                          answered Jan 6 at 22:57









                          Ran KiriRan Kiri

                          13315




                          13315























                              0












                              $begingroup$

                              Another approach uses the mean value theorem for integrals. Indeed, set $g_n(x)=f(x^n).$ Then, there is a sequence $(c_n)subseteq (0,1-epsilon)$ such that $intlimits_0^{1-epsilon}g_n(x)dx=g_n(c_n)(1-epsilon)=f(c^n_n)(1-epsilon).$ Then, $c_n^nto 0$ as $nto infty$ and so $f(c_n^n)to f(0).$



                              For the second part, note that $int_0^1f(x^n),dx=int_0^{1-epsilon}f(x^n),dx+int_{1-epsilon}^1f(x^n),dx,$ so we just need to consider the second integral. But as $g_n$ is bounded, we can apply the dominated convergence theorem to write $lim int_{1-epsilon}^1f(x^n),dx=int_{1-epsilon}^1f(0)dx=epsilon f(0)$.



                              Therefore, $lim int_0^1f(x^n),dx= f(0).$






                              share|cite|improve this answer











                              $endgroup$













                              • $begingroup$
                                why the downvote? since $(c_n)subseteq (0,1-epsilon), limsup c_n^nto 0$ and since $f$ is continuous, $f(c_n^n)to 0$.
                                $endgroup$
                                – Matematleta
                                Jan 6 at 23:16
















                              0












                              $begingroup$

                              Another approach uses the mean value theorem for integrals. Indeed, set $g_n(x)=f(x^n).$ Then, there is a sequence $(c_n)subseteq (0,1-epsilon)$ such that $intlimits_0^{1-epsilon}g_n(x)dx=g_n(c_n)(1-epsilon)=f(c^n_n)(1-epsilon).$ Then, $c_n^nto 0$ as $nto infty$ and so $f(c_n^n)to f(0).$



                              For the second part, note that $int_0^1f(x^n),dx=int_0^{1-epsilon}f(x^n),dx+int_{1-epsilon}^1f(x^n),dx,$ so we just need to consider the second integral. But as $g_n$ is bounded, we can apply the dominated convergence theorem to write $lim int_{1-epsilon}^1f(x^n),dx=int_{1-epsilon}^1f(0)dx=epsilon f(0)$.



                              Therefore, $lim int_0^1f(x^n),dx= f(0).$






                              share|cite|improve this answer











                              $endgroup$













                              • $begingroup$
                                why the downvote? since $(c_n)subseteq (0,1-epsilon), limsup c_n^nto 0$ and since $f$ is continuous, $f(c_n^n)to 0$.
                                $endgroup$
                                – Matematleta
                                Jan 6 at 23:16














                              0












                              0








                              0





                              $begingroup$

                              Another approach uses the mean value theorem for integrals. Indeed, set $g_n(x)=f(x^n).$ Then, there is a sequence $(c_n)subseteq (0,1-epsilon)$ such that $intlimits_0^{1-epsilon}g_n(x)dx=g_n(c_n)(1-epsilon)=f(c^n_n)(1-epsilon).$ Then, $c_n^nto 0$ as $nto infty$ and so $f(c_n^n)to f(0).$



                              For the second part, note that $int_0^1f(x^n),dx=int_0^{1-epsilon}f(x^n),dx+int_{1-epsilon}^1f(x^n),dx,$ so we just need to consider the second integral. But as $g_n$ is bounded, we can apply the dominated convergence theorem to write $lim int_{1-epsilon}^1f(x^n),dx=int_{1-epsilon}^1f(0)dx=epsilon f(0)$.



                              Therefore, $lim int_0^1f(x^n),dx= f(0).$






                              share|cite|improve this answer











                              $endgroup$



                              Another approach uses the mean value theorem for integrals. Indeed, set $g_n(x)=f(x^n).$ Then, there is a sequence $(c_n)subseteq (0,1-epsilon)$ such that $intlimits_0^{1-epsilon}g_n(x)dx=g_n(c_n)(1-epsilon)=f(c^n_n)(1-epsilon).$ Then, $c_n^nto 0$ as $nto infty$ and so $f(c_n^n)to f(0).$



                              For the second part, note that $int_0^1f(x^n),dx=int_0^{1-epsilon}f(x^n),dx+int_{1-epsilon}^1f(x^n),dx,$ so we just need to consider the second integral. But as $g_n$ is bounded, we can apply the dominated convergence theorem to write $lim int_{1-epsilon}^1f(x^n),dx=int_{1-epsilon}^1f(0)dx=epsilon f(0)$.



                              Therefore, $lim int_0^1f(x^n),dx= f(0).$







                              share|cite|improve this answer














                              share|cite|improve this answer



                              share|cite|improve this answer








                              edited Jan 6 at 23:27

























                              answered Jan 6 at 22:49









                              MatematletaMatematleta

                              10.2k2918




                              10.2k2918












                              • $begingroup$
                                why the downvote? since $(c_n)subseteq (0,1-epsilon), limsup c_n^nto 0$ and since $f$ is continuous, $f(c_n^n)to 0$.
                                $endgroup$
                                – Matematleta
                                Jan 6 at 23:16


















                              • $begingroup$
                                why the downvote? since $(c_n)subseteq (0,1-epsilon), limsup c_n^nto 0$ and since $f$ is continuous, $f(c_n^n)to 0$.
                                $endgroup$
                                – Matematleta
                                Jan 6 at 23:16
















                              $begingroup$
                              why the downvote? since $(c_n)subseteq (0,1-epsilon), limsup c_n^nto 0$ and since $f$ is continuous, $f(c_n^n)to 0$.
                              $endgroup$
                              – Matematleta
                              Jan 6 at 23:16




                              $begingroup$
                              why the downvote? since $(c_n)subseteq (0,1-epsilon), limsup c_n^nto 0$ and since $f$ is continuous, $f(c_n^n)to 0$.
                              $endgroup$
                              – Matematleta
                              Jan 6 at 23:16


















                              draft saved

                              draft discarded




















































                              Thanks for contributing an answer to Mathematics Stack Exchange!


                              • Please be sure to answer the question. Provide details and share your research!

                              But avoid



                              • Asking for help, clarification, or responding to other answers.

                              • Making statements based on opinion; back them up with references or personal experience.


                              Use MathJax to format equations. MathJax reference.


                              To learn more, see our tips on writing great answers.




                              draft saved


                              draft discarded














                              StackExchange.ready(
                              function () {
                              StackExchange.openid.initPostLogin('.new-post-login', 'https%3a%2f%2fmath.stackexchange.com%2fquestions%2f3064458%2ffind-lim-limits-n-rightarrow-infty-int-limits-01fxndx%23new-answer', 'question_page');
                              }
                              );

                              Post as a guest















                              Required, but never shown





















































                              Required, but never shown














                              Required, but never shown












                              Required, but never shown







                              Required, but never shown

































                              Required, but never shown














                              Required, but never shown












                              Required, but never shown







                              Required, but never shown







                              Popular posts from this blog

                              An IMO inspired problem

                              Management

                              Investment